Autor Tema: Goldbach según un informático .

0 Usuarios y 1 Visitante están viendo este tema.

24 Enero, 2023, 10:03 am
Respuesta #10

Luis Fuentes

  • el_manco
  • Administrador
  • Mensajes: 56,049
  • País: es
  • Karma: +0/-0
Hola

Gracias por tu respuesta, y aunque parece obvio que la lista 3 tiene menos tues que la 1 , busco si esta demostrado,.
la primera lista tiene numeros primos.
la segunda lista tiene numeros que pueden ser primos para que se incumpla la conjetura de goldbach.
la tercera tiene los primos de la segunda, es decir los primos que no cumplen con la conjetura de goldback
si hay igual numero de primos que no cumplen la conjetura, que primos, la conjetura queda como falsa. caso contrario, como cierta.

No es imposible para cualquier numero que eso sea asi, por ejemplo para los impares que acaban en 7, menos el 7.

Si, bueno me refería a que es obvio que es menor O IGUAL.

Efectivamente si la primera y tercera lista tienen el mismo número de trues, quiere decir que para todo primo \( k \), se tiene que \( n-k \) NO es primo y por tanto la conjetura de Goldbach fallaría.

Si NUNCA tienen el mismo número de trues, significa que existe un primo \( k \) tal que \( n-k \) también es primo, y por tanto la conjetura es cierta.

Pero entonces es una equivalencia trivial con la conjetura de Goldbach; es decir es la conjetura de Goldbach "explicada" con otras palabras. Esto no supone ningún avance para intentar demostrarla.

Saludos.

24 Enero, 2023, 10:13 am
Respuesta #11

vmanalb

  • $$\Large \color{#5372a0}\pi\,\pi$$
  • Mensajes: 82
  • País: es
  • Karma: +0/-0
    • Víctor Manzanares Alberola
Bueno, razonando he llegado a la conclusión que seguro la tercera lista tiene menos trues que la primero si el rango a evaluar el el doble de un numero primo,
porque rango- rango /2 = primo. por lo que tendríamos mínimo un false mas en la columna 3 que en la 1 . Lo cual es una tonteria,.

Pero no se si se puede demostrar para otro tipo de numeros

24 Enero, 2023, 10:16 am
Respuesta #12

Luis Fuentes

  • el_manco
  • Administrador
  • Mensajes: 56,049
  • País: es
  • Karma: +0/-0
Hola

Bueno, razonando he llegado a la conclusión que seguro la tercera lista tiene menos trues que la primero si el rango a evaluar el el doble de un numero primo,
porque rango- rango /2 = primo. por lo que tendríamos mínimo un false mas en la columna 3 que en la 1 .

Pero ahí lo único que estás diciendo es que si un número par es el doble de un número primo se puede escribir (obviamente) como suma de dos primos (el mismo primo sumado consigo mismo).

¿Qué aporta?.

Saludos.

24 Enero, 2023, 10:37 am
Respuesta #13

vmanalb

  • $$\Large \color{#5372a0}\pi\,\pi$$
  • Mensajes: 82
  • País: es
  • Karma: +0/-0
    • Víctor Manzanares Alberola
Bueno, razonando he llegado a la conclusión que seguro la tercera lista tiene menos trues que la primero si el rango a evaluar el el doble de un numero primo,
porque rango- rango /2 = primo. por lo que tendríamos mínimo un false mas en la columna 3 que en la 1 . Lo cual es una tonteria,.

Pero no se si se puede demostrar para otro tipo de numeros
Para un rango en el que hay mas primos que impares esto también es cierto, la columna 3 tiene menos trues que la 1.
No he calculado el máximo de esto, pero seria fácil de contar. aprox.
n/ln(n)=n/2-n/ln(n) o algo asi ,. perdon si me equivoco.

24 Enero, 2023, 11:47 am
Respuesta #14

feriva

  • $$\Large \color{#a53f54}\pi\,\pi\,\pi\,\pi\,\pi\,\pi\,\pi$$
  • Mensajes: 11,330
  • País: es
  • Karma: +1/-0
  • Sexo: Masculino
Hola, vmanalb.

Imagino que lo dices te interesa para pares grandes, puesto que la conjetura se cumple para billones de números pares consecutivos.

Si fuera así, entonces se puede decir lo que sigue:

Dado un par \( 2\cdot n \) muy grande, en el intervalo \( (0,n) \) existe una cantidad de primos que es, con bastante aproximación, el triple de la que hay en el intervalo \( ({\color{green}n},2n){\color{green}} \); (yo no me acuerdo de la demostración ahora, pero es sencilla y, un día que pregunté sobre esto, me lo demostró Luis; si te interesa ver cómo es se lo dices).

Dado que todos los números del intervalo que suman 2n, primos o no, tienen que ser simétricos, uno de ellos pertenecerá a \( (0,n) \) y otro a \( (n,2n) \) o bien será el propio n sumado con él mismo. Por tanto, como la cantidad de primos está desequilibrada (siendo 1/3 en el segundo intervalo) si pensamos solamente en parejas primo-primo quedarán primos “viudos” (primos que sumarán 2n con no primos). En cambio, si consideramos parejas exclusivamente del tipo primo-no primo, no se puede asegurar (hasta que se demuestre) que queden primos “viudos”; si queda alguno, entonces existe al menos una pareja primo-primo. Esto es debido a que la cantidad de no primos es mucho mayor que la de primos y, por consiguiente, no es impedimento para que los primos “casen” todos con no primos.

De hecho, si esta conjetura fuera gobernada por el puro azar, no se cumpliría seguro (creo que con probabilidad 1 de no cumplirse, aunque a lo mejor meto la pata al considerarlo).

El teorema de los números primos nos hace ver que la densidad de los primos va dececiendo de cero a 2n según tomamos pares más grandes (no de forma monótona, pero a tramos va descendiendo). Para números muy grandes la cantidad de primos es muy pequeña respecto de los no primos. Es como tener dos tarros con muchas judías y unos poquitos garbanzos en cada bote mezclados con las judías. Si extraemos al azar una legumbre de cada bote, lo más normal será sacar judía-judía ó judía-garbazo... y más raramente garbanzo-garbanzo. Bien, podemos sacar garbanzo-garbanzo para un par grande, puede pasar; pero es que la conjetura ésta supone hacer el experimento aleatorio con infinitos pares de botes cada vez más grandes, con una densidad de garbanzos cada vez más baja. Así, por probabilidad, en algún momento no sacaremos ninguna pareja de garbanzos; y basta con que falle una pareja de botes para que falle la conjetura.

Sorprendentemente, contra pronóstico, a medida que los pares son más grandes, existen más coincidencias primo-primo; no crecen de forma monótona las parejas, pero van creciendo a tramos. Es decir, llega un momento que la cantidad de parejas siempre está por encima de un número de dos cifras, después por encima de uno de tres; siempre hasta donde se observa... No se sabe si puede descender bruscamente o no, hasta donde se mira no lo parece; ésa es la cuestión (todo esto que digo, lo digo porque lo he comprobado hasta cierto número de pares haciendo programillas en Python, no me lo invento).

Con eso te quiero decir que si estás pensando en una cuestión azarosa relacionada con cantidades de primos para que se cumpla, abandona la idea, que no funciona, esta conjetura no se cumple por suerte (lo que no asegura al 100% que no pueda dejar de cumplirse, por muy raro que se vea el que pueda pasar eso).

Saludos.

24 Enero, 2023, 11:58 am
Respuesta #15

vmanalb

  • $$\Large \color{#5372a0}\pi\,\pi$$
  • Mensajes: 82
  • País: es
  • Karma: +0/-0
    • Víctor Manzanares Alberola
Yo digo que mi experimento de las listas, demuestra que la conjetura se cumple para los numeros menores a e^4.

te voy a volver a leer, porque me ha costado entenderte, muchas gracias.


Que conste que eres un máquina y muchas gracias por tu tiempo

aqui no seria
l teorema de los números primos nos hace ver que la densidad de los primos va dececiendo de cero a 2n según tomamos pares más grandes (no de forma monótona,
l teorema de los números primos nos hace ver que la densidad de los primos va creciendo de cero a 2n según tomamos pares más grandes (no de forma monótona,

24 Enero, 2023, 12:04 pm
Respuesta #16

feriva

  • $$\Large \color{#a53f54}\pi\,\pi\,\pi\,\pi\,\pi\,\pi\,\pi$$
  • Mensajes: 11,330
  • País: es
  • Karma: +1/-0
  • Sexo: Masculino
Yo digo que mi experimento de las listas, demuestra que la conjetura se cumple para los numeros menores a e^4.

te voy a volver a leer, porque me ha costado entenderte, muchas gracias.

aqui no hay un fallo
Dado un par 2⋅n muy grande, en el intervalo (0,n) existe una cantidad de primos que es, con bastante aproximación, el triple de la que hay en el intervalo (0,2n);
seria ?
Dado un par 2⋅n muy grande, en el intervalo (0,n) existe una cantidad de primos que es, con bastante aproximación, el triple de la que hay en el intervalo (n,2n);

Sí, es un despiste al teclear, gracias; ahora lo cambio.

En lo otro no, la densidad de cero a a 2n baja, hay menos primos en comparación con no primos en la medida que 2n es más grande (no de forma monótona, es decir, baja y sube en dientes de sierra, es como una escalera que sube; pero esa subida supone bajada en cuanto a densidad de primos) Bueno, según quieras dibujar la gráfica representando la densidad de los no primo o primos; puede ser una escalera que sube o que baja

Saludos.

24 Enero, 2023, 12:17 pm
Respuesta #17

vmanalb

  • $$\Large \color{#5372a0}\pi\,\pi$$
  • Mensajes: 82
  • País: es
  • Karma: +0/-0
    • Víctor Manzanares Alberola
Yo digo que mi experimento de las listas, demuestra que la conjetura se cumple para los numeros menores a e^4.

te voy a volver a leer, porque me ha costado entenderte, muchas gracias.

aqui no hay un fallo
Dado un par 2⋅n muy grande, en el intervalo (0,n) existe una cantidad de primos que es, con bastante aproximación, el triple de la que hay en el intervalo (0,2n);
seria ?
Dado un par 2⋅n muy grande, en el intervalo (0,n) existe una cantidad de primos que es, con bastante aproximación, el triple de la que hay en el intervalo (n,2n);

Sí, es un despiste al teclear, gracias; ahora lo cambio.

En lo otro no, la densidad de cero a a 2n baja, hay menos primos en comparación con no primos en la medida que 2n es más grande (no de forma monótona, es decir, baja y sube en dientes de sierra, es como una escalera que sube; pero esa subida supone bajada en cuanto a densidad de primos) Bueno, según quieras dibujar la gráfica representando la densidad de los no primo o primos; puede ser una escalera que sube o que baja

Saludos.

Ahora te entiendo entendía otra cosa.

Estoy buscando en internet una demostración teórica del 4 al 54 y no la encuentro, sabrías decirme si existe, o esta solo en este grupo .

24 Enero, 2023, 12:35 pm
Respuesta #18

feriva

  • $$\Large \color{#a53f54}\pi\,\pi\,\pi\,\pi\,\pi\,\pi\,\pi$$
  • Mensajes: 11,330
  • País: es
  • Karma: +1/-0
  • Sexo: Masculino
Yo digo que mi experimento de las listas, demuestra que la conjetura se cumple para los numeros menores a e^4.

te voy a volver a leer, porque me ha costado entenderte, muchas gracias.

aqui no hay un fallo
Dado un par 2⋅n muy grande, en el intervalo (0,n) existe una cantidad de primos que es, con bastante aproximación, el triple de la que hay en el intervalo (0,2n);
seria ?
Dado un par 2⋅n muy grande, en el intervalo (0,n) existe una cantidad de primos que es, con bastante aproximación, el triple de la que hay en el intervalo (n,2n);

Sí, es un despiste al teclear, gracias; ahora lo cambio.

En lo otro no, la densidad de cero a a 2n baja, hay menos primos en comparación con no primos en la medida que 2n es más grande (no de forma monótona, es decir, baja y sube en dientes de sierra, es como una escalera que sube; pero esa subida supone bajada en cuanto a densidad de primos) Bueno, según quieras dibujar la gráfica representando la densidad de los no primo o primos; puede ser una escalera que sube o que baja

Saludos.

Ahora te entiendo entendía otra cosa.

Estoy buscando en internet una demostración teórica del 4 al 54 y no la encuentro, sabrías decirme si existe, o esta solo en este grupo .

Es que eso es un caso particular que no entiendo muy bien para qué quieres inspeccionar; la conjetura se cumple para  los números pares desde 4 hasta no se cuántos billones, todos consecutivos sin que falle ninguno.

Esta cuestión la puedes ver con ejemplos

\( 1,2,3,4,5,6 \): mitad de primos y mitad de no primos 3/6 = 1/2

\( 1,2,3,4,5,6,7,9,10 \) menos de la mitad de primos... 4/10

y así lo vas mirando...

Cuando el par x es muy grande, la cantidad de primos se aproxima a \( \dfrac{x}{log(x)} \), la cantidad de no primos será entonces

\( x-\dfrac{x}{log(x)}=x(1-\dfrac{1}{log(x)}) \).

donde x crece (vamos tomando pares más grandes) y por tanto \( (1-\dfrac{1}{log(x)}) \) también crece, porque se va a acercando de cero, o casi cero, a 1 cada vez más. Es decir, al diferencia crece a favor de los no primos y eso supone baje la densidad de primos.

Saludos.

24 Enero, 2023, 12:37 pm
Respuesta #19

vmanalb

  • $$\Large \color{#5372a0}\pi\,\pi$$
  • Mensajes: 82
  • País: es
  • Karma: +0/-0
    • Víctor Manzanares Alberola
No nos estamos entendiendo .

¿Existe una demostración teórica de que la conjetura por huevos es cierta de 4 al 54??